问题
三维库仑势的形式为:
按动量Fourier展开:
求 $V(\vec{k})$ 。
求解
但 $e^{-ikr}$ 与 $e^{-ikr}$ 在 $+\infty$ 处发散。
使其收敛
为了解决发散问题,可在一开始令
所以
注
函数满足一致收敛,积分与极限才可交换顺序。
类似的变换
用同样的方法, 可以得到
用 Mathematica 可以验证.
Reference
- 关于使其收敛的说明, 分别从物理的角度, 数学的角度阐述这样做的合理性和必要性: https://physics.stackexchange.com/questions/7462/fourier-transform-of-the-coulomb-potential